<< Chapter < Page Chapter >> Page >
y = lim f ( x n ) = lim f ( x n k ) = f ( x ) ,

showing that y f ( C ) , implying that f ( C ) is closed.

This theorem tells us something about the range of a continuous function of a real or complex variable. It says that if a subset of the domain is closed and bounded, so is the image of that subset.

The next theorem is about continuous real-valued functions of a complex variable, and it is one of the theorems to remember.

Let f be a continuous real-valued function on a compact subset S of C . Then f attains both a maximum and a minimum value on S . That is, there exist points z 1 and z 2 in S such that f ( z 1 ) f ( z ) f ( z 2 ) for all z S .

We prove that f attains a maximum value, leaving the fact that f attains a minimum value to the exercise that follows. Let M 0 be the supremum of the set of all numbers f ( x ) for x S . (How do we know that this supremum exists?) We will show that there exists an z 2 S such that f ( z 2 ) = M 0 . This will finish the proof, since we would then have f ( z 2 ) = M 0 f ( z ) for all z S . Thus, let { y n } be a sequence of elements in the range of f for which the sequence { y n } converges to M 0 . (This is Exercise 2.20 again.) For each n , let x n be an element of S such that y n = f ( x n ) . Then the sequence { f ( x n ) } converges to M 0 . Let { x n k } be a convergent subsequence of { x n } . (How?) Let z 2 = lim x n k . Then z 2 S , because S is closed, and f ( z 2 ) = lim f ( x n k ) , because f is continuous. Hence, f ( z 2 ) = M 0 , as desired.

  1. Prove that the f of the preceding theorem attains a minimum value on S .
  2. Give an alternate proof of [link] by using [link] , and then proving that a closed and bounded subset of R contains both its supremum and its infimum.
  3. Let S be a compact subset of C , and let c be a point of C that is not in S . Prove that there is a closest point to c in S . That is, show that there exists a point w S such that | w - c | | z - c | for all points z S . HINT: The function z | z - c | is continuous on the set S .

Let f : [ a , b ] R be a real-valued function that is continuous at each point of [ a , b ] .

  1. Prove that the range of f is a closed interval [ a ' , b ' ] . Show by example that the four numbers f ( a ) , f ( b ) , a ' and b ' can be distinct.
  2. Suppose f is 1-1. Show that, if c is in the open interval ( a , b ) , then f ( c ) is in the open interval ( a ' , b ' ) .

We introduce next a different kind of continuity called uniform continuity. The difference between regular continuity and uniform continuity is a bit subtle,and well worth some thought.

A function f : S C is called uniformly continuous on S if for each positive number ϵ , there exists a positive number δ such that | f ( x ) - f ( y ) | < ϵ for all x , y S satisfying | x - y | < δ .

Basically, the difference between regular continuity and uniform conintuity is that the same δ works for all points in S .

Here is another theorem worth remembering.

A continuous complex-valued function on a compact subset S of C is uniformly continuous.

We argue by contradiction. Thus, suppose f is continuous on S but not uniformly continuous. Then, there exists an ϵ > 0 for which no positive number δ satisfies the uniform continuity definition. Therefore, thinking of the δ 's as ranging through the numbers 1 / n , we know that for each positive integer n , there exist two points x n and y n in S so that

  1.   | y n - x n | < 1 / n , and
  2.   | f ( y n ) - f ( x n ) | ϵ .

Otherwise, some 1 / n would suffice for a δ . Let { x n k } be a convergent subsequence of { x n } with limit x . By (1) and the triangle inequality, we deduce that x is also the limit of the corresponding subsequence { y n k } of { y n } . But then f ( x ) = lim f ( x n k ) = lim f ( y n k ) , implying that 0 = lim | f ( y n k ) - f ( x n k ) | , which implies that | f ( y n k ) - f ( x n k ) | < ϵ for all large enough k . But that contradicts (2), and this completes the proof.

Continuous functions whose domains are not compact sets may or may not be uniformly continuous, as the next exercise shows.

  1. Let f : ( 0 , 1 ) R be defined by f ( x ) = 1 / x . Prove that f is continuous at each x in its domain but that f is not uniformly continuous there. HINT:Set ϵ = 1 , and consider the pairs of points x n = 1 / n and y n = 1 / ( n + 1 ) .
  2. Let f : [ 1 , ) [ 1 , ) be defined by f ( x ) = x . Prove that f is not bounded, but is nevertheless uniformly continuous on its domain. HINT: Take δ = ϵ .

Let f : S T be a continuous 1-1 function from a compact (closed and bounded) subset of C onto the (compact) set T . Let g : T S denote the inverse function f - 1 of f . Then g is continuous. The inverse of a continuous function, that has a compact domain, is also continuous.

We prove that g is continuous by using [link] ; i.e., we will show that g - 1 ( A ) is closed whenever A is a closed subset of C . But this is easy, since g - 1 ( A ) = g - 1 ( A S ) = f ( A S ) , and this is a closed set by [link] , because A S is compact. See part (e) of [link] .

REMARK Using the preceding theorem, and the exercise below, we will show that taking n th roots is a continuous function. that is, the function f defined by f ( x ) = x 1 / n is continuous.

Use the preceding theorem to show the continuity of the following functions.

  1. Show that if n is an odd positive integer, then there exists a continuous function g defined on all of R such that g ( x ) is an n th root of x for all real numbers x . That is, ( g ( x ) ) n = x for all real x . (The function f ( x ) = x n is 1-1 and continuous.)
  2. Show that if n is any positive integer then there exists a unique continuous function g defined on [ 0 , ) such that g ( x ) is an n th root of x for all nonnegative x .
  3. Let r = p / q be a rational number. Prove that there exists a continuous function g : [ 0 , ) [ 0 , ) such that g ( x ) q = x p for all x 0 ; i.e., g ( x ) = x r for all x 0 .

Let f be a continuous 1-1 function from the interval [ a , b ] onto the interval [ c , d ] . Then f must be strictly monotonic, i.e., strictly increasing everywhere or strictly decreasing everywhere.

Since f is 1-1, we clearly have that f ( a ) f ( b ) , and, without loss of generality, let us assume that c = f ( a ) < f ( b ) = d . It will suffice to show that if α and β belong to the open interval ( a , b ) , and α < β , then f ( α ) f ( β ) . (Why will this suffice?) Suppose by way of contradiction that there exists α < β in ( a , b ) for which f ( α ) > f ( β ) . We use the intermediate value theorem to derive a contradiction. Consider the four points a < α < β < b . Either f ( a ) < f ( α ) or f ( β ) < f ( b ) . (Why?) In the first case ( f ( a ) < f ( α ) ), f ( [ a , α ] ) contains every value between f ( a ) and f ( α ) . And, f ( [ α , β ] ) contains every value between f ( α ) and f ( β ) . So, let v be a number such that f ( a ) < v , f ( β ) < v , and v < f ( α ) (why does such a number v exist?). By the Intermediate Value Theorem, there exists x 1 ( a , α ) such that v = f ( x 1 ) , and there exists an x 2 ( α , β ) such that v = f ( x 2 ) . But this contradicts the hypothesis that f is 1-1, since x 1 x 2 . A similar argument leads to a contradiction in the second case f ( β ) < f ( b ) . (See the following exercise.) Hence, there can exist no such α and β , implying that f is strictly increasing on [ a , b ] .

Derive a contradiction from the assumption that f ( β ) < f ( b ) in the preceding proof.

Get Jobilize Job Search Mobile App in your pocket Now!

Get it on Google Play Download on the App Store Now




Source:  OpenStax, Analysis of functions of a single variable. OpenStax CNX. Dec 11, 2010 Download for free at http://cnx.org/content/col11249/1.1
Google Play and the Google Play logo are trademarks of Google Inc.

Notification Switch

Would you like to follow the 'Analysis of functions of a single variable' conversation and receive update notifications?

Ask